Entrada destacada

Distancia media de dos puntos en un cuadrado unitario

martes, 9 de abril de 2019

Problema de Optimización por Multiplicadores de Lagrange: con restricción no acotada


Enunciado del Problema

Optimizar la función f (x, y, z) = x² + y² + z² sujeto a las restricciones g(x,y,z) = x - y = 1 y
h(x,y,z) = y² - z² = 1.


solución


Por Multiplicadores de Lagrange se tiene que los puntos óptimos, si existe, satisfacen el sistema de ecuaciones generado por:



que corresponde :



Resolviéndolo tenemos :

Solve[{2 x == la, 2 y == -la + 2 mu y, 2 z == -2 mu z, y^2 - z^2 == 1, x - y == 1}, {x, y, z, la, mu}, Reals]




Es decir, obtenemos dos puntos (0,-1,0) y (2,1,0), al calcularlos en f, tenemos:

f[x_, y_, z_] := x^2 + y^2 + z^2
{f[0, -1, 0], f[2, 1, 0]}

{1,5}

Graficando las dos restricciones :

h1 = ContourPlot3D[1 == y^2 - z^2, {x, -5, 5}, {y, -5, 5}, {z, -5, 5},ContourStyle -> {Red, Opacity[0.2]}, Mesh -> None];
h2 = ContourPlot3D[1 == x - y, {x, -5, 5}, {y, -5, 5}, {z, -5, 5}, 
   ContourStyle -> {Blue, Opacity[0.2]}, Mesh -> None];
Show[h1, h2]




La curva restricción, junto con los puntos óptimos:

h3 = ParametricPlot3D[{{t + 1, t, Sqrt[t^2 - 1]}, {t + 1, 
     t, -Sqrt[t^2 - 1]}}, {t, 1, 10}, 
   PlotStyle -> {{Thickness[0.015], Red}}];
h4 = ParametricPlot3D[{{t + 1, t, Sqrt[t^2 - 1]}, {t + 1, 
     t, -Sqrt[t^2 - 1]}}, {t, -10, -1}, 
   PlotStyle -> {{Thickness[0.015], Red}}];
h5 = Graphics3D[{{Yellow, PointSize[0.03], Point[{0, -1, 0}]}, {Green,
      PointSize[0.03], Point[{2, 1, 0}]}}];
Show[h1, h2, h3, h4, h5]



Determinando puntos sobre cada una de las ramas de la curva restricción y calculándolos en f,
primera rama:

Table[{t + 1, t, Sqrt[t^2 - 1]}, {t, 1, 10, 1}]





f @@@ %

{5, 16, 33, 56, 85, 120, 161, 208, 261, 320}

Segunda rama:

Table[{t + 1, t, Sqrt[t^2 - 1]}, {t, -10, -1, 1}]





f @@@ %

{280, 225, 176, 133, 96, 65, 40, 21, 8, 1}

vemos que el 1 y el 5 son los valores mínimos sobre cada uno de las ramas de la curva restricción, es decir hemos encontrado son mínimos locales:

Utilizando la función Minimize, tenemos:

Minimize[{x^2 + y^2 + z^2, y^2 - z^2 == 1, x - y == 1}, {x, y, z}]



y con restricciones sobre y, por cada rama:

Minimize[{x^2 + y^2 + z^2, y^2 - z^2 == 1, x - y == 1, y > 0}, {x, y, z}]



Minimize[{x^2 + y^2 + z^2, y^2 - z^2 == 1, x - y == 1, y < 0}, {x, y, z}]



Determinando el máximo total:

Maximize[{x^2 + y^2 + z^2, y^2 - z^2 == 1, x - y == 1}, {x, y, z}]



Al no estar acotada la curva restricción, tenemos mínimos para cada rama pero no existe un máximo.

Graficando junto la función f :

Manipulate[
 Show[ContourPlot3D[
   x^2 + y^2 + z^2 == r, {x, -4, 4}, {y, -4, 4}, {z, -4, 4},
   Mesh -> 2, PlotPoints -> 10, ContourStyle -> Opacity[0.2]], h3, h4,
   h5], {r, 0, 10}]




Para aprender más sobre Mathematica ingrese aquí sitio de aprendizaje de Wolfram o en mi website ustamathematica.wixsite.com/basicas


No hay comentarios.:

Publicar un comentario